1
$\begingroup$

I am reading Arthur's book "Introductionto the trace formula".

In reading the book, two small question has arised and so I would like to ask it.

  1. Let $G$ be a connected reductive group over $\mathbb{Q}$ and $P=M_PN_P$ a standard parabolic subgroup. (here $M_P$ is Levi subgroup and $N_P$ is the unipotent subgroup of $P$)

I am wondering whether $G(\mathbb{Q})$ acts on $N_P(\mathbb{A})$. Because it looks that Arthur used such fact in some argument.

Is it really true?

  1. Let $N$ be an arbitrary unipotent group defined over $\mathbb{Q}$ which has $G$-action over $\mathbb{Q}$. Denote $G$-action by $\rho \colon G \to Aut(N)$.

The for arbitrary $g\in G(\mathbb{A})$, let $n'=\rho(g^{-1})(n)$. Then I heard that two measures $dn$ and $dn'$ has the relation $dn'=\delta_{\rho}(g)dn$ for some character $\delta_{\rho} \colon G(\mathbb{A}) \to \mathbb{C}^{\times}$.

I am wondering the explicit formula for the character $\delta_{\rho}$.

For these two questions, any comments will be greatly appreciated.

$\endgroup$
10
  • 2
    $\begingroup$ Could you be more precise about how Arthur seems to use this? At first glance, I would say that the action should be of $M_P(\mathbb{Q})$ on $N_P(\mathbb{A})$, not $G$. For example, if $G=\mathrm{GL}(2)$ and $P=B$ is the Borel, there is no natural action of $G$ on $N\cong \mathbb{A}$. $\endgroup$ Aug 20, 2019 at 20:05
  • $\begingroup$ @WSL, In page 76 of Arthur's book, Arthur wrote "The kernel $K_{P,\chi}(x,y)$ defined in Sec.12 is invariant under left translation of either variable by $N_P(\mathbb{A})$". I think this is the part where Arthur used the fact $G(\mathbb{Q})$ acts on $N_P(\mathbb{A})$. Without this, can we verify this sentence? $\endgroup$
    – Monty
    Aug 21, 2019 at 10:48
  • $\begingroup$ The kernel under consideration is given as a sum of (integrals of) Eisenstein series induced from $P$, and is formally invariant under $N_P$ as these Eisenstein series are. He is not claiming that the entire automorphic kernel $K_f$ is invariant, which is not true. $\endgroup$ Aug 21, 2019 at 11:40
  • $\begingroup$ @WSL, Why Eisenstein series are left $N_P(\mathbb{A})$ invariant? It is not obvious from the definition of Eisensterin series. $\endgroup$
    – Monty
    Aug 21, 2019 at 12:39
  • $\begingroup$ Ah you’re right: I was being sloppy and thinking about the representation. Still, I expect that the definition for the kernel he gives will solve your question. Can you post the formula from Section 12? $\endgroup$ Aug 21, 2019 at 12:45

1 Answer 1

1
$\begingroup$

I will try to answer both your questions in the context of Arthur's notes.

There does not exist a canonical action of a general connected reductive group $G$ on $N_P$ where $P = N_P M_P$ is a parabolic subgroup. One however looks at the exact sequence $$ 1 \to N_P \to P \to M_P \to 1 $$ which gives an action of $M_P$ on $N_P$ by conjugation.

Suppose $\text d n$ is a Haar measure on the unimodular group $N_P(\mathbb A)$. For every $m \in M_P(\mathbb A)$, we have another measure on $N_P(\mathbb A)$, namely $\text d(mnm^{-1})$. By the uniqueness of Haar measures, there is a positive number $\delta_m$ such that $\text d(mnm^{-1}) = \delta_m \text dn$. It is not difficult to show that the map $m \mapsto \delta_m$ is a homomorphism of groups. There must thus be a character $\delta : M_P(\mathbb A) \to \mathbb R^*_{> 0}$ which we call the modulus character.

This modulus character plays an important role in the theory of automorphic forms. For instance, Arthur uses this when making a change of variables involving the truncated kernel. (If I recall correctly, Cartier's notes in the Corvallis proceedings talk about the modulus character).

Given $P = M_P N_P$, there is an explicit formula for this modulus character $\delta = \delta_{M_P}$. It is a good exercise to show that this equals half of the sum of positive roots of $(P, A_P)$ where $A_P$ is the split part of the center of $M_P$. Try this out when $G = SL(2)$. Perhaps look at the Lie algebra picture.

$\endgroup$
1
  • $\begingroup$ Thank you Abhi. I am studying Arthur’s book these days. Your comments are really helpful to me. Thank you very much!😊 $\endgroup$
    – Monty
    Sep 1, 2019 at 17:38

Your Answer

By clicking “Post Your Answer”, you agree to our terms of service and acknowledge that you have read and understand our privacy policy and code of conduct.

Not the answer you're looking for? Browse other questions tagged or ask your own question.